LSAT and Law School Admissions Forum

Get expert LSAT preparation and law school admissions advice from PowerScore Test Preparation.

User avatar
 Dave Killoran
PowerScore Staff
  • PowerScore Staff
  • Posts: 5862
  • Joined: Mar 25, 2011
|
#27189
Complete Question Explanation
(The complete setup for this game can be found here: lsat/viewtopic.php?t=11429)

The correct answer choice is (B)


Irrespective of the Local conditions, answer choice (C) can be eliminated by applying the Not Laws.

Since neither F nor N is scheduled, it follows that H, L, O, P, S, and W must be scheduled. And since F is not scheduled, P cannot be scheduled for day 1 or day 2 (the contrapositive of the final rule). Consequently P must be scheduled for day 3 and thus answer choice (D) is incorrect. Since O is scheduled for day 2 and P is scheduled for day 3, the third rule can be applied, and it follows that since L cannot be scheduled with O or P, then L must be scheduled for day 1. Thus, answer choice (B) is correct.

Get the most out of your LSAT Prep Plus subscription.

Analyze and track your performance with our Testing and Analytics Package.